Search found 50 matches


Re: Massive confusion over exponents at GMAT...

[quote="quant-master"][quote="gmat_2010"]Hi, What does x^y^z mean? Does it mean (x^y)^z or x^(y^z). All throughout my school this question k!IIed me and I always got the right answers when I went for x^(y^z). But in the test prep companies' questions the answer is generally (x^y)...

by gmat_2010

Tue Aug 11, 2009 12:34 pm
Forum: Problem Solving
Topic: Massive confusion over exponents at GMAT...
Replies: 3
Views: 1318

Massive confusion over exponents at GMAT...

Hi, What does x^y^z mean? Does it mean (x^y)^z or x^(y^z). All throughout my school this question killed me and I always got the right answers when I went for x^(y^z). But in the test prep companies' questions the answer is generally (x^y)^z = x^(y*z). Today, I got a similar question while taking a ...

by gmat_2010

Tue Aug 11, 2009 11:24 am
Forum: Problem Solving
Topic: Massive confusion over exponents at GMAT...
Replies: 3
Views: 1318

Shouldn't it be "pay" and not "pay for" here. You "pay" your bills, cost, etc. You "pay for" a service, a sandwich, any benefit by "paying" its cost. Just wondering. I would have'd gone for A as it has both "demanding that" (I guess "d...

by gmat_2010

Mon Aug 03, 2009 9:20 am
Forum: Sentence Correction
Topic: subjunctive question
Replies: 12
Views: 2441

This is how this Bernoulli's Trial came: You need exactly k nos. of success in n trials. Hence, you need k success nos. and n - k nos. of failures, all should occur together. Probability of success in a single trial is p. Hence probability of failure is 1-p Hence, probability is (p^k)*((1-p)^(n-k)) ...

by gmat_2010

Sun Aug 02, 2009 2:17 pm
Forum: Problem Solving
Topic: simple but tricky
Replies: 10
Views: 6143

I get 1/3 too. Consider the two keys as "one". Now, since we have 5 spaces in the chain, the "one" key can be arranged in the chain in 5 ways. Now, each "one" key can be in the orientation, AB or BA. Hence, for each 5 slots above, AB and BA are possible. Hence, 10 ways ...

by gmat_2010

Sun Aug 02, 2009 1:55 pm
Forum: Problem Solving
Topic: probability
Replies: 18
Views: 4633

I thought it was B. The below link confirms the same: http://www.novapress.net/diagnostic/lsat_test/arg_test_sol.html Here are my reasons. Economy can collapse because of many factors. One of them is failure in GNP increment by 20%. But, if GNP increases by 20%, there is 100% probability that GNP CA...

by gmat_2010

Sun Aug 02, 2009 3:57 am
Forum: Critical Reasoning
Topic: logic-must be true
Replies: 11
Views: 2480

I don't understand why the concentration decrease over a period of time. The premise states that the plant produces histadine. Hence, I don't get it. Unless...... one implies that the plants eventual death is not because of age but because of increase in toxic metal content.

by gmat_2010

Sat Aug 01, 2009 5:44 pm
Forum: Critical Reasoning
Topic: this one is dicy
Replies: 17
Views: 3129

Yup! D. Anything to do with the spice, is not important here as we have to find out the benefits of New Amsterdam WITHOUT considering the nutmeg story. D is more credible than E. This is not a CR question. This is an RC. I remember attending to this passage when I was taking a test (don't remember w...

by gmat_2010

Sat Aug 01, 2009 5:02 pm
Forum: Critical Reasoning
Topic: NutMeg Trade
Replies: 11
Views: 4927

But, statement 1 doesn't state that r is not equal to 5. Hence, E.

by gmat_2010

Fri Jul 31, 2009 1:03 pm
Forum: Data Sufficiency
Topic: Value of r
Replies: 3
Views: 1302

I think it is E. In any case it can't be C because condition 2 can be derived from condition 1. Consider x intercept = a, y intercept = b Then, equation of line is: (x/a) + (y/b) = 1 Or, y = (-b/a)*x + b Also, from 1: slope is 3 times the y-intercept Hence, (-b/a) = 3*b Or, a = -1/3 So, C is wrong.

by gmat_2010

Fri Jul 31, 2009 4:59 am
Forum: Data Sufficiency
Topic: What is the y-intercept?
Replies: 16
Views: 15617

Yes! Range = max in the set - min in the set

by gmat_2010

Fri Jul 31, 2009 4:50 am
Forum: Data Sufficiency
Topic: very tough ds question from OG 12
Replies: 4
Views: 1410

Let f be the fee. Let n be the number of people. 0.75% less fee = 0.9925*f Hence, by A: 0.9925*f*(n+100) = n*f f cancels out as f<>0. Therefore, 0.9925*n + 99.75 = n 0.0075*n = 99.75 n = 13300 Hence A is sufficient. For B: (f + 1.5)*(n - 100) = f*n This yields an equation with two unknowns. Hence, B...

by gmat_2010

Fri Jul 31, 2009 4:20 am
Forum: Data Sufficiency
Topic: fee
Replies: 10
Views: 5610

Can B be the answer? Since we are calculating HCF and LCM I assume a, b, c, d are integers. (Can negative numbers have HCF and LCM, I don't know. Can anyone tell?) So, LCM of a and b will be >= to (greater of a and b). HCF of c and d will be <= to (smaller of c and d). Since, LCM = HCF: a + b < = 2 ...

by gmat_2010

Fri Jul 31, 2009 3:39 am
Forum: Data Sufficiency
Topic: DS problem
Replies: 4
Views: 2035

For the second statement:

m - n > n
Therefore, m > 2n
Therefore, m/n < 2 (sign changes because we know n is negative).
Hence, all we've got is -- m/n is less than 2. Hence, not B but E.

by gmat_2010

Fri Jul 31, 2009 2:39 am
Forum: Data Sufficiency
Topic: Inequalities
Replies: 10
Views: 1798

I am guessing he has assumed x^2y as (x^2)y. I assumed it as x^(2y). If it is (x^2)y then he is correct.

by gmat_2010

Fri Jul 31, 2009 2:32 am
Forum: Data Sufficiency
Topic: xy equation
Replies: 4
Views: 1483